LSAT and Law School Admissions Forum

Get expert LSAT preparation and law school admissions advice from PowerScore Test Preparation.

 Administrator
PowerScore Staff
  • PowerScore Staff
  • Posts: 8916
  • Joined: Feb 02, 2011
|
#37024
Please post below with any questions!
 Margo
  • Posts: 18
  • Joined: Jun 25, 2018
|
#47786
I got this answer incorrect--I chose A--but I think I understand why this is incorrect.

The conclusion states that for a meeting to have max productivity, it needs to have a clear time frame and be no more than 30 min long.

I diagrammed this as:

If achieve max productivity :arrow: clear time frame & no more than 30 min long

Is A incorrect because it is a mistaken reversal--it assumes that just because a meeting has a clear time frame and is no more than 30 min long, it is automatically maximally productive?

I can see that C is correct because it states that a meeting at the company will be maximally productive ONLY IF it has a clear time frame and lasts no more than 30 min. ("Only if" introduces the necessary condition here)
When I diagram C I got:
If maximum productivity :arrow: clear time frame & no more than 30 min long

So, my diagrams for the conclusion and C are the same. Am I thinking about this the right way? Hope this post makes sense.
Thanks!
 Adam Tyson
PowerScore Staff
  • PowerScore Staff
  • Posts: 5153
  • Joined: Apr 14, 2011
|
#48060
Absolutely correct, Margo! The stimulus uses the word "needs" to indicate the necessary condition, and your diagram of that claim is perfect. A is indeed a Mistaken Reversal of that claim, while C is a restatement of that claim. Well done!

Get the most out of your LSAT Prep Plus subscription.

Analyze and track your performance with our Testing and Analytics Package.